Sample test questions

Krok 2

()

Терапевтичний профiль 2

1. A 25-year-old woman has been A. Transfer into the inpatient suffering from diabetes mellitus since she department was 9. She was admitted into the B. Continue the treatment in the therapeutic unit with significant edemas of the , , department and legs. Blood pressure - 200/110 mm Hg, C. Transfer into the neuroresuscitation Hb- 90 g/L, blood creatinine - 850 mcmol/L, department urine proteins - 1.0 g/L, leukocytes - 10-15 in D. Compulsory medical treatment for the vision field. Glomerular filtration rate - alcoholism 10 mL/min. What tactics should the doctor E. Discharge from the hospital choose? 5. After eating shrimps, a 25-year-old man A. Transfer into the hemodialysis unit suddenly developed skin itching, some areas B. Active conservative for diabetic of his skin became hyperemic or erupted into nephropathy vesicles. Make the diagnosis: C. Dietotherapy D. Transfer into the clinic A. Acute urticaria E. Renal transplantation B. Hemorrhagic vasculitis (Henoch-Schonlein purpura) 2. A 59-year-old woman was brought into the C. Urticaria pigmentosa unit. Extremely severe case D. Psoriasis of scleroderma is suspected. Objectively she E. Scabies presents with malnourishment, ”mask-like” face, and acro-osteolysis. Blood: erythrocytes 6. A 25-year-old woman complains of fatigue, - 2.2 · 109/L, erythrocyte sedimentation rate - dizziness, hemorrhagic rashes on the skin. 40 mm/hour. Urine: elevated levels of free She has been presenting with these signs for a · 12 oxyproline. Name one of the most likely month. Blood test: erythrocytes - 1.0 10 /L, pathogenetic links in this case: Hb- 37 g/L, color index - 1.1, leukocytes - 1.2·109/L, platelets - 42 ·109/L. What analysis A. Formation of antibodies to collagen would be the most advisable for diagnosis- B. Formation of antibodies to native DNA making in this case? C. Formation of antibodies to blood corpuscles A. Sternal puncture (bone marrow biopsy) D. Formation of antibodies to transversely B. Splenic biopsy striated muscles C. Liver biopsy E. Formation of antibodies to vessel wall D. Coagulation studies E. US of the gastrointestinal tract 3. A 34-year-old man on the 3rd day of ceftriaxone treatment for acute otitis (daily 7. A 35-year-old man complains of rapidly dosage - 2 grams) developed diarrhea incresing fatigue, palpitations, ”visual snow”, occurring 5-6 times per day.Feces are without and dizziness. He has a history of peptic mucus or blood admixtures. Temperature is ulcer of the stomach. Objectively the skin 36.6oC. Gregersen reaction (occult blood in is pale. Vesicular respiration is observed in feces) is negative. Stool culture detected no the lungs. Systolic murmur is detected over pathogenic germs. What is the most likely the cardiac apex, heart rate is 100/min., BP cause of diarrhea in this case? is 100/70 mm Hg. The is slightly tender on palpation. Blood test: erythrocytes A. Antibiotic-associated diarrhea - 3.2 · 1012/L, Нb- 100 g/L, color index - B. Intestinal dysbiosis 0.95. What type of anemia is the most likely C. Bacterial overgrowth syndrome present in this case? D. Ulcerative colitis E. Crohn’s disease (regional enteritis) A. Posthemorrhagic anemia B. Sideroblastic anemia 4. A chronic alcoholic was hospitalized C. Chronic iron-deficiency anemia into the therapeutic inpatient unit due D. Hemolytic anemia to pneumonia. On the day 5 of his E. Hypoplastic anemia hospitalization he became disoriented in time and space, developed fear-inducing 8. A 62-year-old patient has been visual hallucinations and motor agitation. hospitalized with complaints of pain in Full body tremor and tremor of the limbs are the on the right during breathing, observed. X-ray and physical examinations dyspnea, and dry cough. Ten days ago he detect the signs of his convalescence from slipped and fell hitting his right side. On pneumonia. What tactics should be chosen examintaion: the patient lies on the left regarding this patient? side. The right side of the thorax lags during breathing. On the right there are crepitation and pain in the III-IV ribs. Dullness of percussion sound and sharply diminished breath sounds can be observed. On X-ray: Терапевтичний профiль 3 signs of exudate, fracture of the III-IV ribs. A. Acute left ventricular failure On pleurocentesis: blood is detected. Choose B. Paroxysmal tachycardia the further tactics: C. Bronchial asthma attack D. Pulmonary embolism A. Transfer to a thoracic department E. Acute right ventricular failure B. Prescribe conservative therapy C. Perform repeated pleural taps 12. A 26-year-old woman is suspected to D. Apply a fixation bandage to the rib cage suffer from systemic lupus erythematosus E. Refer to a traumatologist due to systemic lesions of skin, vessels, joints, serous tunics, and heart that developed 9. A 51-year-old man complains of vomiting after photosensitization. The following is with blood. He has been drinking alcohol detected: LE cells, antibodies to native excessively. Health disorder has been DNA, isolated anti-centromere antibodies, observed since he was 40, when he first rheumatoid factor is 1:100, Wassermann developed jaundice. On examination the reaction is positive, circulating immune skin and visible mucosa are icteric, with complex is 120 units. What immunological a stellate vascular pattern. The patient is indicators are considered to be specificto malnourished and presents with abdominal this disease? distension, umbilical hernia, and ascites. The edge of the liver is tapered and painless, +3 A. DNA antibodies cm, the spleen is +2 cm. Blood test: Hb- 80 B. Rheumatoid factor g/L, leukocytes - 3·109/L, platelets - 85·109/L. C. Anti-centromere antibodies What is the cause of portal hypertension in D. Immunoglobulin A this patient? E. Increased circulating immune complex A. Hepatic cirrhosis 13. A woman came to the doctor with complaints of increased body temperature up B. Thrombosis of the splenic vein o C. Hemochromatosis to 37.8 C and moderately sore throat for the D. Constrictive pericarditis last 3 days. Objectively: mandibular lymph E. Budd-Chiari syndrome nodes are enlarged up to 3 cm. Palatine tonsils are hypertrophied and covered with 10. A young man has made an appointment gray coating that spreads to the uvula and with the dermatologist. He complains of a anterior pillars of the fauces. What is the painful facial rash in the beard and mustache most likely diagnosis? area. This condition has been persisting for several weaks already. After shaving, A. Oropharyngeal diphtheria the patient’s condition aggravates. The B. Infectious mononucleosis diagnosis of sycosis is made. What primary C. Pseudomembranous (Vincent’s) tonsillitis morphological elements can be observed in D. Agranulocytosis the rash in this case? E. Oropharyngeal candidiasis A. Pustules, papulae 14. A 42-year-old man, a dispatcher, suffes B. Nodes, nodules from peptic ulcer disease of the duodenum. C. Pustules, bumps The disease is of moderate severity. He wants D. Phlyctenae, maculae to be assigned a disability group. Make the E. Maculae, nodes conclusion regarding his working ability: 11. At night a 63-year-old woman suddenly A. Capable of working, employable developed an asphyxia attack. She has a 15- B. Capable of working, non-employable year-long history of essential hypertension C. First group of disability and had a myocardial infarction 2 years ago. D. Second group of disability Objectively her position in bed is orthopneic, E. Third group of disability the skin is pale, the patient is covered with 15. A 58-year-old man complains of cold sweat, acrocyanosis is observed. Pulse weakness and tumor-like formations that - 104/min. Blood pressure - 210/130 mm appeared on the anterior surface of his Hg, respiration rate - 38/min. Pulmonary and in the inguinal region. Palpation detects percussion sound is clear, with slight dullness soft painless mobile cervical and inguinal in the lower segments; throughout the lymph nodes up to 2 cm in diameter. The lungs single dry crackles can be heard that liver protrudes by 2 cm from the edge of the become bubbling and non-resonant in the costal margin, the lower splenic pole is at lower segments. What is the most likely the umbilical level. In blood: erythrocytes complication in this patient? - 3.5 · 1012/L, Hb- 88 g/L, leukocytes - 86 · 109/L, band neutrophils - 1%, segmented neutrophils - 10%, lymphocytes - 85%, eosinophils - 2%, basocytes - 0%, monocytes - 2%, erythrocyte sedimentation rate - 15 Терапевтичний профiль 4 mm/hour, Gumprecht shadows. What is the has been lasting for 2 years. Objectively: most likely diagnosis? reduced spinal mobility, painful sacroiliac joint, erythrocyte sedimentation rate - 45 A. Chronic lymphatic leukemia mm/hour. X-ray shows narrowing of the B. Lymphocytic leukemoid reaction intervertebral disc space and of the sacroiliac C. Acute leukemia joint. What eye is often associated D. Chronic myeloleukemia with this type of disease progression? E. Lymphogranulomatosis A. Iridocyclitis 16. A 23-year-old man complains of facial B. Retinal detachment edemas, headache, dizziness, low urinary C. Cataract output, and urine discoloration (dark red). D. Optic nerve atrophy These complaints arose after a case of acute E. Blepharitis tonsillitis. On examination there are facial edemas, the skin is pale, temperature is 20. A 63-year-old man complains of 37.4oC; heart rate is 86/min., blood pressure unmotivated weakness and pressing and is 170/110 mm Hg. Heart sounds are muffled, bursting sensation in the left subcostal the II heart sound is accentuated over the area. According to him, these signs aorta. What etiological factor is the most have been present for a year already. likely in this case? Previously he was healthy. He took part in containment measures during the accident A. Beta-hemolytic streptococcus at the Chornobyl Nuclear Power Plant. B. Staphylococcus aureus Objectively: the skin is pale, peripheral C. Streptococcus viridans lymph nodes are not enlarged, the liver D. Streptococcus pyogenes is +3 cm, the spleen is +10 cm. Complete E. Staphylococcus saprophyticus blood count: erythrocytes - 3.1 · 1012/L, · 9 17. An 18-year-old young man complains of Hb- 100 g/L, leukocytes - 46 10 /L, blasts pain in his knee and elbow joints and body - 2%, promyelocytes - 10%, myelocytes - temperature up to 39.5oC. One week and 18%, band neutrophils - 27%, segmented a half earlier he developed sore throat. On neutrophils - 10%, lymphocytes - 12%, examination his body temperature is 38.5oC. eosinophils - 6%, basocytes - 3%, monocytes Swelling of the knee and elbow joints is - 2%, erythrocyte sedimentation rate - 20 observed. Pulse is 106/min., rhythmic. Blood mm/hour. What is the most likely diagnosis? pressure is 90/60 mm Hg. Cardiac borders A. Chronic myeloleukemia are unchanged, heart sounds are weakened, B. Hepatic cirrhosis at the cardiac apex there is a soft systolic C. Acute leukemia murmur. What factor would be the most D. Hemolytic anemia indicative of the likely disease etiology? E. Chronic lymphatic leukemia A. Anti-streptolysin O 21. For three years a 31-year-old woman has B. C-reactive protein been complaining of pain and swelling of C. Creatine kinase her radiocarpal and metacarpophalangeal D. Rheumatoid factor articulations and their reduced mobility in E. Seromucoid the morning, which persisted up to 1.5 hours. 18. A woman has been provisionally Two weeks ago she developed pain, swelling, and reddening of her knee joints, her body diagnosed with pheochromocytoma. At the o stage of intermission her BP is within norm; temperature increased up to 37.5 C.The there is a tendency towards tachycardia. No treatment was untimely. Examination of urine . The decision has been the internal organs revealed no pathologic made to perform a provocative test with alterations. Diagnosis of rheumatoid arthritis histamine. What drug should be kept close was made. What changes are most likely to at hand for emergency aid in case of positive be visible on the arthrogram? test result? A. Joint space narrowing, usuration A. Phentolamine B. Joint space narrowing, subchondral B. Pipolphen (Promethazine) osteosclerosis C. Nifedipine C. Cysts in the subchondral bone Numerous marginal osteophytes Mesaton (Phenylephrine) D. D. E. Epiphyseal osteolysis E. Prednisolone 22. A 52-year-old woman has been 19. A 40-year-old man with Bekhterev suffering for 2 years from dull, occasionally disease (ankylosing spondylitis) complains of o exacerbating pain in her right subcostal elevated body temperature up to 37.8 C,back area, occurring after eating high-fat foods, pain and stiffness, especially observed during bitter taste in her mouth in the morning, the second half of the night. This condition constipations, and flatulence. Objectively she Терапевтичний профiль 5 has excess weight, her body temperature muscles. What is the most likely diagnosis? is 36.9oC; there is a coating on the root of her tongue; the is moderately A. Trigeminal neuralgia distended and painful in the area of B. Glossopharyngeal neuralgia gallbladder projection. What examination C. Temporomandibular joint arthritis would be the most helpful for diagnosis- D. Facial migraine making? E. Maxillary sinusitis A. Ultrasound 26. A 28-year-old man complains of skin B. Duodenal intubation rash and itching on the both of his hands. C. Cholecystography The condition persists for 1.5 years. The D. Duodenoscopy exacerbation of his condition he ascribes to E. Liver scanning the occupational contact with formaldehyde resins. Objectively the lesion foci are 23. A 57-year-old woman complains of symmetrically localized on both hands. weakness, dyspnea, loss of appetite, and Against the background of erythema with liquid feces. She has been suffering from blurred margins there are papulae, vesicles, this condition for 2 years. Objectively erosions, crusts, and scales. What is the most she presents with pale skin, subicteric likely pathology? sclerae, and bright-red fissured tongue. Lymph nodes are not enlarged. Pulse A. Occupational eczema - 100/min. BP- 105/70 mm Hg. Liver B. Idiopathic eczema +3 cm, the spleen cannot be palpated. C. Allergic dermatitis Blood test: erythrocytes - 1.2 · 1012/L, D. Simple contact dermatitis Нb- 56 g/L, color index - 1.4, macrocytes, E. Erythema multiforme · 9 leukocytes - 2, 5 10 /L, eosinophils - 1%, 27. A 20-year-old student after failing an juvenile - 1%, metamyelocytes - 1%, band exam developed complaints of a sensation of neutrophils - 8%, segmented neutrophils - a round foreign body in her throat, difficult 47%, lymphocytes - 38%, monocytes - 4%, swallowing. She fixates on her condition, reticulocytes - 0.1%, platelets - 100 · 109/L, limits her diet, often cries, seeks attention, ESR- 30 mm/hour, indirect bilirubin - 26 exhibits demonstrative attitude. She is highly mmol/L. What changes can be expected in susceptible to psychotherapeutic suggestion. the bone marrow puncture material? What psychiatric diagnosis can be made in this case? A. Prevalence of megaloblasts B. Increased number of sideroblasts A. Hysterical neurosis C. Erythroid hyperplasia B. Hypochondriacal neurosis D. Presence of blast cells C. Depressive neurosis E. Prevalence of lymphoid tissue D. Obsessive neurosis E. Paranoid personality disorder 24. A 35-year-old man suffers from insulin- dependent diabetes mellitus and chronic 28. A woman with atopic bronchial asthma cholecystitis. He takes NPH insulin: 20 units was found to have one allergen to dog in the morning and 12 units in the evening. hair +++. Carpets were removed from the After a meal he developed pain in the right apartment, the apartment was renovated, subcostal area, nausea, vomiting, sleepiness, and air conditioner was installed. However, and increased polyuria. What prehospital recurrent asphyxia attacks still occur every measures will be the most effective for night, despite the patient undergoing prevention of crisis within the next several pathogenetic therapy. What long-term hours? treatment tactics can help this patient to decrease her sensitivity to the allergen? A. Change insulin regimen B. Take analgesics A. Specific hyposensitization C. Take cholagogues B. Continuation of prior treatment D. Exclude fats from the diet C. Antihistamine therapy E. Decrease carbohydrates in the diet D. Buteyko breathing technique E. Referral for speleotherapy 25. A 45-year-old woman complains of intolerable paroxysmal facial pain on the left 29. A 20-year-old man was hospitalized on with attacks that last for 1-2 minutes. Attacks the 9th day of the disease. He attributes are provoked by chewing. The disease onset his disease to eating of insufficiently was two months ago after the overexposure thermally processed pork. At its onset to cold. Objectively: pain at the exit points this condition manifested as periorbital of the trigeminal nerve on the left. Touching edemas and fever. Objectively his body near the wing of the nose on the left induces temperature is 38.5 C. The face is puffy and a pain attack with tonic spasm of the facial the eyelids are markedly swollen. Palpation Терапевтичний профiль 6 of gastrocnemius muscles is sharply painful. exercise test at 50 W there is a depression Blood test shows hypereosinophilia. What is of S-T segment by 3 mm below the isoline in the etiology of this disease? V3-V4. What is the provisional diagnosis? A. Trichinella A. Exertional angina pectoris, functional class B. Trichuris III C. Ascarididae B. Exertional angina pectoris, functional class D. Echinococci IV E. Leptospira C. Exertional angina pectoris, functional class II 30. A 40-year-old man claims that his wife D. Somatoform autonomic dysfunction, is cheating on him and presents a ”proof” hypertension type of her infidelity. He repeatedly initiated E. Alcoholic myocardiodystrophy scandals with his wife at home and at work, demanding that she confess her infidelity, 34. A 45-year-old man, a farmer, presents insulted her, and threatened to kill her. What with acute onset of a disease. He complains preventive measures should be taken against of headache, high temperature, pain in the socially dangerous actions on his part? gastrocnemius muscles, icteric face, and dark urine. Objectively: body temperature - 38oC, A. Consultation with the psychiatrist blood pressure - 100/70 mm Hg, conjunctival B. Outpatient treatment hemorrhages, hepatosplenomegaly, and C. Consultation with the general practitioner oliguria. What is the most likely provisional D. Consultation with the psychologist diagnosis? E. Family counseling A. Leptospirosis 31. A 55-year-old woman complains of B. Brucellosis pain and popping sounds in her left knee C. Viral hepatitis joint, which occur when she climbs the D. Pseudotuberculosis stairs. Occasionally during movements her E. Trichinosis joint becomes ”stuck”. 5 years ago she suffered a trauma of her left knee. Complete 35. A woman has been working as a polisher blood count and biochemical blood analysis for a year and a half. Her workstation show normal results. X-ray shows marked is equipped with a grinding machine osteosclerosis and osteophytes. The joint (grinding wheels). She complains of white space is narrowed. Make the provisional discoloration of her fingers and toes that diagnosis: appears when she is nervous. Objectively there are no changes in the coloration of the A. Osteoarthritis distal segments of her limbs. Grip strength B. Rheumatoid arthritis measured with a dynamometer is 25 kg, C. Gouty arthritis algesimetry findings are 0.1; 0.3; 0.5. Cold D. Psoriatic arthritis stimulus is extremely positive on the upper E. Reactive arthritis and lower limbs. Internal organs are without pathologies. Make the diagnosis: 32. A 40-year-old man, a welder, uses manganese electrodes in his line of work A. Vibration disease (18 years of experience). He complains of B. Raynaud disease difficulties with writing, bad mood, inertness, C. Syringomyelia gait abnormalities, problems with speech, D. Raynaud syndrome and hand tremors. Objectively the following E. Polyneuritis is observed in the patient: hypomimia, increased muscle tone of plastic type, and 36. A 37-year-old man suddenly developed quiet monotonous speech, tremor of the acute headache accompanied by nausea, tongue, pill-rolling tremor of the fingers, and vomiting, and impaired consciousness. retropulsion. What syndrome developed in Objectively blood pressure is 190/120 this patient due to manganese poisoning? mm Hg, the face is hyperemic. Patient’s consciousness is clouded, his answers to A. Parkinsonism the questions are short, monosyllabic. B. Meningism Movement and sensory disturbances are C. Hypothalamic syndrome absent. Meningeal signs are positive. D. Polyneuritic syndrome Cerebrospinal fluid contains blood. What E. Vestibular syndrome provisional diagnosis can be made? 33. A 45-year-old man developed constricting retrosternal pain that occurs during walks at the distance of 200 m. Objectively: heart rate is 80/min., BP is 160/90 mm Hg. During cardiopulmonary Терапевтичний профiль 7

A. Subarachnoid hemorrhage A. Leukocyturia, gross hematuria B. Meningitis B. Gross hematuria C. Ischemic stroke C. Increased blood creatinine and blood urea D. Encephalitis D. Daily proteinuria under 3.0 E. Cerebral vascular embolism E. Daily proteinuria over 3.0 37. A woman undergoing in-patient 41. A 62-year-old woman was brought treatment for viral hepatitis type B into the admission room with complaints developed headache, nausea, recurrent of severe burning retrosternal pain and vomiting, memory lapses, flapping tremor asphyxia. She has a 10-year-long history of her hands, and rapid pulse. Sweet of essential hypertension. Objectively her smell from her mouth is detected. Body condition is moderately severe. She presents temperature is 37.6oC, heart rate is 89/min. with skin pallor, cyanotic , and vesicular What complication developed in the patient? respiration over her lungs. The II heart sound is accentuated over the aorta. Blood pressure A. Acute liver failure - 210/120 mm Hg, heart rate (pulse) - 76/min. B. Ischemic stroke ECG shows elevation of ST segment in the C. Gastrointestinal hemorrhage leads I, AVL, and V5-V6. What is the most D. Hypoglycemic shock likely diagnosis? E. Meningoencephalitis A. Hypertensive crisis complicated with acute 38. A 43-year-old man, a coal-face worker myocardial infarction with 15-year-long record of work, complains B. Uncomplicated hypertensive crisis of cough, thoracic pain, and dyspnea. The C. Hypertensive crisis complicated with cough is mild, usually dry, occurs mostly instable angina pectoris in the morning. The pain is localized in D. Hypertensive crisis complicated with acute the interscapular region and aggravates left ventricular failure during a deep intake of breath. Dyspnea E. Pulmonary embolism occurs during physical exertion. Vesicular respiration in the lungs is weakened. Heart 42. A 35-year-old patient developed an sounds are rhythmic, heart rate is 86/min., epileptic attack with tonoclonic spasms that blood pressure is 135/80 mm Hg. The lasted for 3 minutes. After the attack the abdomen is soft and painless. X-ray shows patient fell asleep but in 5 minutes the second micronodular pulmonary fibrosis. Make the attack occurred. The first step of emergency provisional diagnosis: aid would be to: A. Carboconiosis A. Ensure patency of airways B. Byssinosis B. Take blood from the vein for analysis C. Siderosis C. Introduce diazepam intravenously D. Berylliosis D. Prescribe antiepileptic drugs E. Metal pneumoconiosis E. Administer chloral hydrate via an enema 39. Having examined a 52-year-old patient, 43. A 27-year-old woman, a teacher in the the doctor diagnosed him with obesity (body elementary school, complains of frequent mass index - 34 kg/m2, waist circumference stools, up to 3 times per day, with lumpy - 112 cm) and arterial hypertension (170/105 feces and large amount of mucus, abdominal mm Hg). 2-hour postprandial blood sugar pain that gradually abates after a defecation, is 10.8 mmol/L. What biochemical blood irritability. Her skin is pale and icteric. Pulse analysis needs to be conducted to diagnose is 74/min., rhythmic, can be characterized the patient with metabolic syndrome X? as satisfactory. Blood pressure is 115/70 mm Hg. The abdomen is soft, moderately tender A. Lipid profile along the colon on palpation. Fiberoptic B. Bilirubin colonoscopy detects no changes. What C. Calcium and phosphorus disease can be suspected? D. Creatinine and urea E. Electrolytes A. Irritable bowel syndrome B. Chronic non-ulcerative colitis 40. After overexposure to cold a 45- C. Chronic enteritis year-old woman developed acute pain in D. Crohn disease (regional enteritis) her suprapubic and lumbar areas during E. Whipple disease urination, sharp pains at the end of urination, false urges to urinate. Urine is turbid with 44. A 72-year-old man complains of lower blood streaks. The doctor suspects urinary extremity edema, sensation of heaviness in tract infection. What results of laboratory the right subcostal area, dyspnea at rest. analysis would be the most indicative of such For over 25 years he has been suffering infection? from COPD. Objectively: orthopnea, jugular venous distention, diffuse cyanosis, Терапевтичний профiль 8 acrocyanosis. Barrel chest is observed, on treatment tactics: percussion there is a vesiculotympanitic (bandbox) resonance, sharply weakened A. Pericardial puncture (pericardiocenthesis) vesicular respiration on both sides, moist B. Diuretics crepitant crackles in the lower segments C. Antibiotics of the lungs. Heart sounds are weakened, D. Pericardectomy the II heart sound is accentuated over the E. Glucocorticosteroids pulmonary artery. The liver is +3 cm. What complicated the clinical course of COPD in 48. A 39-year-old man suffers from chronic this patient? rheumatic heart disease. He complains of dyspnea during physical exertion, cough with A. Chronic pulmonary heart expectoration, and palpitations. Ausculation B. Pulmonary embolism detects intensified I heart sound and diastolic C. Acute left ventricular failure murmur; the sound of opening mitral valve D. Diffuse pneumosclerosis can be auscultated at the cardiac apex. E. Community-acquired pneumonia The II heart sound is accentuated over the pulmonary artery. The patient is cyanotic. 45. A 72-year-old man with pneumonia X-ray shows dilated pulmonary root and complains of marked dyspnea, chest pain, enlargement of the right ventricle and left severe cough with expectoration, to is atrium. What is the most likely diagnosis? 39.5-40oC, no urination for a whole day. Objectively the patient is conscious. A. Mitral stenosis Respiratory rate is 36/min. Over the right B. Aortic stenosis lower pulmonary lobe percussion sound C. Pulmonary artery stenosis is dull; on auscultation there is bronchial D. Coarctation of the aorta respiration and numerous moist crackles. E. Patent ductus arteriosus Blood pressure is 80/60 mm Hg. Heart rate is 120/min. Heart sounds are muffled, there is 49. A 23-year-old man complains of severe tachycardia. What tactics should the family pain in his left knee joint. Objectively the doctor choose in the management of this left knee joint is enlarged, with hyperemic patient? skin, painful on palpation. Complete blood count: erythrocytes - 3.8 · 1012/L, Hb- 122 A. Hospitalization into the intensive care unit g/L, leukocytes - 7.4 · 109/L, platelets - B. Outpatient treatment 183 · 109/L. Erythrocyte sedimentation rate C. Treatment in the day patient facility - 10 mm/hour. Bleeding time (Duke method) D. Hospitalization into the unit - 4 min., Lee-White coagulation time - 24 E. Hospitalization into the unit min. Partial thromboplastin time (activated) 46. 2 hours after eating unknown - 89 seconds. Rheumatoid factor - negative. mushrooms, a 28-year-old man sensed a What is the most likely diagnosis? decrease in his mobility and deterioration of A. Hemophilia, hemarthrosis his ability to focus. This condition was then B. Werlhof disease (immune followed by a state of agitation and agression. On examiantion he is disoriented and his thrombocytopenia) speech is illegible. 4 hours later he developed C. Rheumatoid arthritis fetor hepaticus and lost his consciousness. D. Thrombocytopathy What syndrome can be observed in this E. Hemorrhagic vasculitis (Henoch-Schonlein patient? purpura), articular form A. Acute hepatic failure 50. A 24-year-old woman, a kindergarten B. Hepatolienal syndrome teacher, has been sick for 2 days already. Disease onset was acute. She presents with C. Portal hypertension o D. Cholestatic syndrome elevated body temperature up to 38.0 C, E. Cytolytic syndrome pain attacks in her lower left abdomen, liquid stool in small amounts with blood and mucus 47. A 36-year-old man complains of marked admixtures 10 times a day. Pulse - 98/min., dyspnea and cardiac pain. He ascribes his blood pressure - 110/70 mm Hg. Her tongue disease to the case of influenza that he had is moist and coated with white deposits. The 2 weeks ago. Objectively he leans forward abdomen is soft, the sigmoid colon is painful when sitting. The face is swollen, cyanotic, and spastic. Make the provisional diagnosis: cervical veins are distended. Heart borders are extended on the both sides, heart sounds A. Shigellosis are muffled, heart rate = Ps = 118/min., B. Escherichiosis BP is 90/60 mm Hg. Blood test: ESR is C. Salmonellosis 46 mm/hour. ECG shows low voltage. X- D. Yersiniosis ray shows trapezoidal cardiac silhouette and E. Rotavirus infection signs of pulmonary congestion. Choose the Терапевтичний профiль 9

51. A 38-year-old woman complains of tuberculosis clinic. Throughout the last 3 weakness, sleepiness, pain in the joints, weeks he has been suffering from headaches weight gain despite low appetite, and of increasing intensity. Neurological constipations. She presents with dry and examination detects nuchal rigidity without thickened skin, puffy and amimic face, focal signs. Make the provisional diagnosis: narrowed palpebral fissures, thick tongue, and deep hoarse voice. Her heart sounds are A. Tuberculous meningitis weak, pulse is 56/min. Low levels of free T4 B. Chorea minor are observed. This patient needs to take the C. Brain tumor following on a regular basis: D. Myelitis E. Convexital arachnoiditis A. Thyroxine B. Mercazolil (Thiamazole) 56. A patient has gradually lost C. Lithium carbonate consciousness. The skin is pale and dry.There D. Furosemide is a smell of ammonia from the mouth. E. Calcium gluconate Respirations are deep and noisy. Heart sounds are muffled, pericardial friction rub 52. A 23-year-old man has accidentally is present. Blood pressure is 180/130 mm swallowed brake fluid. After that he has been Hg. Blood test: Нb- 80 g/L, leukocytes - presenting with anuria for 5 days already; 12 · 109/L, blood glucose - 6.4 mmol/L, urea - his creatinine levels elevated up to 0.569 50 mmol/L, creatinine - 1200 mcmol/L, blood mmol/L. What treatment tactics should be osmolarity - 350 mOsmol/L. No urinary chosen in this case? excretion. Make the diagnosis: A. Hemodialysis A. Uremic coma B. Detoxication therapy B. Hyperglycemic coma C. Antidotal therapy C. Acute renal failure D. Diuretics D. Acute disturbance of cerebral circulation E. Plasmapheresis E. Hyperosmolar coma 53. A 52-year-old man for the last 3 years 57. A 72-year-old man diagnosed with has been suffering from difficult swallowing ischemic heart disease presents with diffuse of solid food, burning retrosternal pain cardiosclerosis, permanent tachysystolic that aggravated during eating, loss of atrial fibrillation, heart failure IIа, FC III. body mass, and occasional vomiting with Objective examination of vital signs: blood undigested food. Esophageal X-ray shows pressure is 135/80 mm Hg, heart rate is S-shaped deformation of the esophagus 160/min., pulse is 125/min. Left ventricular and its dilation; at the cardiac orifice the ejection fraction is 32%. What drug is esophagus is constricted; esophageal mucosa indicated in this case and should be presribed is smooth, without signs of peristalsis. Make to the patient? the provisional diagnosis: A. Digoxin A. Esophageal carcinoma B. Procainamide (Novocainamide) B. Diaphragmatic hernia C. Isadrine (Isoprenaline) C. Esophageal achalasia D. Verap ami l D. Reflux esophagitis E. Ivabradine E. Esophageal diverticulum 58. A 34-year-old man complains of pale 54. A 53-year-old man complains of edema of the face, feet, shins, and lumbar general weakness, loss of appetite, and area, elevated blood pressure up to 160/100 painful vesicles appearing on his skin. mm Hg, and general weakness. He has The disease onset occurred suddenly, after a clinical history of nonspecific ulcerative hyperinsolation one week ago. Examination colitis. Objectively: pulse - 84/min., rhythmic, detects isolated vesicles with wrinkled blood pressure - 165/100 mm Hg; edemas all opercula and occasional painful erosions on over the body; the skin is pale and dry, with the skin of the patient’s torso and limbs. low turgor. The kidneys cannot be palpated, Nikolsky sign is positive. What is the most on an attempt to palpate them they are likely diagnosis? painless. Blood test: erythrocytes - 3.0·1012/L, A. Acantholytic pemphigus Нb- 100 g/L, erythrocyte sedimentation B. Nonacantholytic pemphigus rate - 50 mm/hour. Urinalysis: proteins - 3.5 g/L, erythrocytes - 7-10 in the vision C. Duhring’s disease (dermatitis field, leukocytes - 5-6 in the vision field. herpetiformis) Daily proteinuria - 6 grams. What analysis D. Herpes should be conducted additionally to verify E. Toxicodermia the diagnosis? 55. A patient is being treated in the Терапевтичний профiль 10

A. Gingival biopsy for the diagnosis of blood circulation. Oblectively her pulse is amyloid disease 96/min., of unequal strength. Blood pressure B. Radioisotopic examination of kidneys is 110/70 mm Hg, heart rate is 120/min. ECG C. Urinalysis for Bence-Jones protein registers small unevenly-sized waves in place D. Renal ultrasound of P-waves, R-R intervals are of unequal E. Survey and excretory urography length. What is the most likely diagnosis? 59. A 42-year-old man, a worker at the A. Atrial fibrillation meat processing factory, developed an B. Paroxysmal supraventricular tachycardia itching spot on his lower jaw, which C. Atrial flutter gradually transformed into a slightly painful D. Paroxysmal ventricular tachycardia carbuncle 3 cm in diameter, surrounded by E. Respiratory arrhythmia a painless swelling that reaches the clavicle. Temperature is subfebrile, under 37.8oC.The 63. An 18-year-old patient always obeys doctor suspects anthrax. What drug should others and adapts his needs to the demands this man be prescribed for treatment? of the people on whom he depends. He excessively defers to their wishes and makes A. Penicillin them responsible for his wellbeing, cannot B. Levomycetin (Chloramphenicol) defend his interests and needs support C. Biseptol (Co-trimoxazole) from other people. Such psychic profile D. Interferon alpha has been formed in the childhood, remains E. Azidothymidin (Zidovudine) unchanged, and hinders adaptation. What psychic disorder is observed in this patient? 60. A 57-year-old patient complains of dyspnea at rest. The patient presents with A. Dependent personality disorder orthopnea, acrocyanosis, bulging cervical B. Anxiety (avoidant) personality disorder veins. On percussion: dull sound over the C. Anankastic personality disorder lower lung segments. On auscultation: no D. Markedly accentuated personality respiratory sounds. Heart rate is 92/min. E. Psychopathy-like state Right-sided cardiac dilatation is observed. The liver is +7 cm. Shins are swollen. Pleural 64. A 45-year-old man with thrombophlebitis effusion is suspected. What indicator would of the deep veins in his legs suddenly after confirm the presence of transudate in this physical exertion developed sharp pain in his case? thorax on the right, dyspnea, and hemoptysis. Objectively his condition is severe; he A. Total protein content in the pleural fluid presents with acrocyanosis, shortening of below 25 g/L pulmonary percussion sound on the right, B. Presence of atypical cells and weakened respiration. Respiration is C. Total protein content in the pleural fluid 30/min., blood pressure is 110/80 mm Hg. exceeding 30 g/L ECG shows sinus tachycardia, heart rate is D. Specific gravity exceeding 1015 120/min., electrical axis of the heart deviates E. Positive Rivalta’s test to the right, SI -QIII. What is the most likely diagnosis? 61. A 33-year-old man developed multiple rashes on the skin of his torso and extensor A. Pulmonary embolism surfaces of his upper and lower limbs. B. Community-acquired right-sided The rashes itch and occasionally fuse pneumonia together and form plaques. The elements C. Cancer of the right lung of rash are covered with silver-white fine D. Right-sided exudative pleurisy scales that easily flake off when scratched. E. Spontaneous pneumothorax Grattage test results in three sequential phenomena: stearin spot, terminal film, and 65. A 48-year-old woman has been punctate hemorrhage. What diagnosis can be hospitalized due to development of suspected? tachysystolic atrial fibrillation. She has lost 5 kg of body weight within 2 months. On A. Psoriasis palpation there is a node in the left lobe of B. Parapsoriasis the thyroid gland. What pathology resulted C. Pyoderma in the development of this condition? D. Lichen ruber planus E. Secondary papular syphilid A. Toxic nodular goiter B. Aterosclerotic cardiosclerosis 62. A 38-year-old woman after physical C. Chronic thyroiditis overexertion suddenly developed palpitations, D. Nontoxic nodular goiter dyspnea, and a dull pain in the cardiac E. Autoimmune thyroiditis area. For 10 years she has been registered for regular check-ups due to rheumatism 66. A 48-year-old woman developed and mitral valve disease with non-disturbed insomnia, depressive mood, anxiety, fears Терапевтичний профiль 11 and suicidal thoughts after the death of A. β-blockers her husband that occurred one month ago. B. Angiotensin-converting enzyme inhibitors During her stay in the hospital she speaks C. Diuretics in a low voice, is depressed, anxious, avoids D. Calcium antagonists sleeping, refuses to eat. What medications E. Imidazoline receptor antagonists should be prescribed in this case? 71. A 45-year-old woman is registered for A. Antidepressants regular check-ups due to Werlhof disease B. Antipsychotics (immune thrombocytopenia). Complete C. Group B vitamins blood count: Нb- 100 g/L, erythrocytes - D. Nootropics 2.8 · 1012/L, platelets - 90.0 · 109/L, leukocytes E. Anticonvulsants - 8.4 · 109/L, erythrocyte sedimentation rate - 13 mm/hour. Examination detects a single 67. A 32-year-old woman complains of small hematoma on the anterior surface episodes of intense fear that occur without of the thigh, developed after the patient visible cause and last for 10-20 minutes; the accidentally stumbled on a table. What episodes are characterized by rapid pulse, treatment tactics should be chosen in this sweating, labored breathing, and vertigo. case? Specify the likely diagnosis: A. Continue the supervision by the hospital A. Panic disorder hematologist B. Paranoid syndrome B. Urgent hospitalization into the C. Manic syndrome unit D. Simple schizophrenia C. Urgently start a hemostatic therapy, E. Claustrophobia followed by a planned hospitalization into the 68. A 39-year-old man suffers from hematology unit chronic adrenal insufficiency and receives D. Urgent hospitalization into the general replacement glucocorticoid therapy care unit (hydrocortisone - 15 mg/day). He is to E. Administer thrombocytic mass, continue undergo elective surgery for calculous the treatment in the hematology unit cholecystitis. What medication adjustment 72. The dermatologist has an appointment should be made on the day of the surgery with a 30-year-old man that complains to prevent the development of acute adrenal of severely itching rashes that especially insufficiency? disturb him at night. The rashes developed 2 weeks ago, after he had returned from a A. Increase the dosage by 2-3 times Cancelthedrugforthedayofthesurgery travel. Objectively on the lateral surfaces B. of his fingers, hands, wrists, elbows, lower C. Add a mineralocorticoid Addanantibiotic abdomen, genitals, and thighs there are D. paired papulovesicles, single pustules, E. Prescribe a large volume intravenous fluid infusion and scratch marks. What disease can be suspected? 69. After a long drive with the window open a man developed facial asymmetry; A. Scabies he cannot close his right eye, his right B. Pyoderma nasolabial fold is smoothed out, movements C. Dermatitis D. Eczema of expression are absent on the right, there Shingles is a disturbance of gustatory sensation in the E. tongue on the right. No other neurological 73. A 38-year-old woman developed a pathologies were detected. What disease can medical condition 7 days after her return be provisionally diagnosed in this patient? from Bangladesh. Periodical elevation Neuropathy of the facial nerve of temperature was accompanied by A. chills and excessive sweating. She was B. Neuropathy of the trigeminal nerve Trigeminal ganglionitis diagnosed with tropical malaria. Next day C. her condition further deteriorated: body D. Neuropathy of the oculomotor nerve oC Ischemic stroke temperature - 38 , inertness, periodical E. loss of consciousness, generalized seizures, 70. A 56-year-old woman was diagnosed with tachycardia, hypotension, and icteric skin. stage 2 hypertension of the 2nd degree. She What complication can be suspected in this belongs to the group of moderate risk and case? has bronchial asthma. What group of drugs is CONTRAINDICATED to this patient? Терапевтичний профiль 12

A. Cerebral coma creatorrhea. What prescription would be the B. Serous meningitis most advisable in this case? C. Purulent meningitis D. Acute hepatic failure A. Multi-enzyme preparations E. Acute heart failure B. Cholinergic antagonists C. Metronidazole and loperamide 74. A 73-year-old woman came to the family D. Antacids and antispasmodics for one of her regular follow-up E. Cholinergic antagonists and antibacterial examinations. Three months ago she was agents found to have type 2 diabetes mellitus. She was keeping to her diet and exercise 77. A man was brought into the admission plan and taking phytopreparations. On room after an overexposure to cold. He examination her fasting glucose was within complains of sharp pain in the small of the range of 7.8-8.6 mmol/L, HbА1с - 7.9%. his back and elevated body temperature Height - 164 cm, weight - 83 kg. What up to 38oC. He took some aspirin. Blood blood sugar-controlling medicine should test: leukocytes - 10.5 · 1012/L, eosinophils she be prescribed first in the course of her - 5%, band neutrophils - 8%, segmented pharmacological therapy? neutrophils - 51%, lymphocytes - 32%, monocytes - 4%, erythrocyte sedimentation A. Metformin rate - 28 mm/hour. Urinalysis: protein - 0.6 B. Glibenclamide g/L, leukocytes - cover the whole vision field, C. Glimepiride large amount of mucus. What is the most D. Gliclazide likely diagnosis? E. Insulin A. Acute pyelonephritis 75. A 27-year-old man complains of pain in B. Chronic pyelonephritis his leg joints, purulent discharge from the C. Acute glomerulonephritis eyes, and painful burning sensations during D. Tubulointerstitial nephritis urination. Disease onset was acute. He has a E. Subacute malignant glomerulonephritis history of influenza. The patient smokes and drinks alcohol in excess. In his line of work 78. A 26-year-old man complains of he is often away on business trips. What is the chills, rhinitis, dry cough, and fever up to most likely etiological factor of this disease? 38oC. Examination shows him to be in a moderately severe condition; there are A. Chlamydia small pale pink non-merging spots on the B. Adenovirus skin of his back, abdomen, and extremities. C. Streptococci Palpation reveals enlarged occipital and D. Staphylococci axillary lymph nodes. No information about E. Candida vaccination history could be obtained. What 76. A 46-year-old woman has diarrhea with is the likely etiology of this disease? abdominal distension, loss of body mass, A. Rubella virus and large amounts of porridge-like foul- B. Epstein-Barr virus smelling stool without blood streaks or C. Streptococcus tenesmus. Objective examination detects D. Mumps virus moderate tenderness in the mesogastrium E. Neisseria meningitis and left abdominal flank. Feces analysis detects steatorrhea with neutral fat and Хiрургiчний профiль 13

1. During medical examination a cadet A. Frostbite of the I degree in the naval college was detected to have B. Perniosis a painless dense ulcer 1.5x0.5 in size in his C. Frostbite of the II degree perianal area at the 2 o’clock position. The D. Frostbite of the III degree ulcer floor resembles ”old fat”. What is the E. Frostbite of the IV degree provisional diagnosis? 5. A 16-year-old patient has made an A. Hard syphilitic chancre of the rectum appointment with an otolaryngologist. He B. Rectal fissure complains of elevated body temperature C. Rectal fistula and sore throat. Disease onset was 2 days D. Anal cancer ago, after the patient ate two portions of E. Anal crypt suppuration ice-cream. Pharyngoscopy shows hyperemic mucosa of the palatine tonsils, with purulent 2. A 32-year-old woman complains of tumor- exudate in the lacunae. Make the provisional like formation on the anterior surface of her diagnosis: neck that appeared 2 years ago. Within the last 3 months the tumor has been rapidly A. Lacunar tonsillitis growing. It hinders swallowing and impairs B. Follicular tonsillitis speech; the tumor causes a sensation of C. Diphtheria pressure. Objectively the skin moisture is D. Acute pharyngitis normal, pulse is 80/min., rhythmic, blood E. Pseudomembranous (Vincent’s) tonsillitis pressure is 130/80 mm Hg. In the right lobe of the thyroid gland there is a dense lumpy node 6. A 35-year-old woman complains of high 3.0x3.5 cm that moves during swallowing. body temperature and pain in the upper Scanning image shows a ”cold nodule” in outer quadrant of her right buttock, which the thyroid gland. Make the provisional developed after an injection. She has been diagnosis: presenting with this condition for 3 days. At the site of injection the skin is hyperemic; A. Thyroid cancer there is a painful infiltrate with an area B. Thyroid adenoma of softening in its center. The woman is C. Thyroid cyst diagnosed with a postinjection abscess of D. Nodular goiter the right buttock. What tactics should the E. Autoimmune thyroiditis surgeon choose in this case? 3. After a surgery for a left thigh phlegmon A. Abscess incision, sanation and drainage of the disease progression was complicated by the cavity sepsis. On the 7th day after the surgery B. Hospitalization, prescription of antibiotics, there are marked signs of a generalized UHF inflammatory reaction, in blood there are C. Abscess puncture, pus removal followed by signs of toxic anemia and progressing application of antiseptics hypoproteinemia, bilirubin levels are 40 D. 10-15 minutes of low-intensity laser mcmol/L, AST and ALT exceed the norm radiation directed at the right buttock by 2.5 times. Oliguria persists (700 mL of E. Antipyretic agents, massage, and urine per day). Name the phase of sepsis application of dry heat to the right buttock progression: 7. A 65-year-old woman on abdominal A. Catabolic phase palpation presents with a tumor in the B. Stress phase and above it; the tumor is C. Anabolic phase 13x8 cm in size, moderately painful, non- D. Recovery phase mobile, pulsing. On auscultation systolic E. Mixed phase murmur can be observed. What is the most likely diagnosis? 4. A 10-year-old boy, who was outdoors in windy and cold weather, developed moderate A. Abdominal aortic aneurysm pain and tingling in his fingers and toes. B. Gastric tumor When he returned home, his parents noticed C. Arteriovenous aneurysm that the tips of his fingers and toes were white D. Tricuspid insufficiency and their sensitivity was lost. As the affected E. Bicuspid insufficiency areas were warming up, the fingers and toes developed tingling and painful sensations. 8. A 32-year-old man complains of pain Skin pallor changed into redness, tingling in his legs that intensifies during walking, stopped, mild itching and swelling of the intermittent claudication, numbness of his fingers appeared. Determine the frostbite toes, extremity coldness, and inability to walk degree in this child: more that 100 meters. When he sleeps, his leg usually hangs down. The patient has been smoking since he was 16. He drinks alcohol in excess. The left leg is colder than the right Хiрургiчний профiль 14 one; the skin of the extremities is dry. No burns of the 15% of the body surface. On the pulse can be detected on the pedal arteries, 20th day after the trauma the patient presents while pulsation of the femoral arteries is with sharp increase of body temperature, retained. What is the most likely diagnosis? general weakness, rapid vesicular respiration; facial features are sharpened, BP is 90/50 mm A. Obliterating endarteritis Hg, heart rate is 112/min. What complication B. Diabetic angiopathy is it? C. Leriche syndrome (aortoiliac occlusive disease) A. Sepsis D. Raynaud disease B. Pneumonia E. Deep thrombophlebitis C. Acute intoxication D. Purulent bronchitis 9. A 50-year-old patient was brought to E. Anaerobic infection a hospital with complaints of blood in urine. Urination is painless and undisturbed. 13. 2 hours after a traffic accident a 28-year- Macrohematuria had been observed for old man in a grave condition was brought 3 days. Objectively: kidneys cannot be to a hospital. The patient complains of palpated, suprapubic area is without abdominal pain. He received a blow to the alterations, external genitalia are non- abdomen with the steering wheel. Objective pathologic. On rectal investigation: prostate examination revealed the following: the is not enlarged, painless, has normal abdomen does not participate in respiration, structure. Cystoscopy revealed no changes. is tense and acutely painful on palpation; What is the most likely diagnosis? the abdominal muscles are defensively tense, peritoneal irritation signs are positive, A. Renal carcinoma hepatic dullness is absent. BP is 90/60 mm B. Bladder tuberculosis Hg, heart rate is 120/min. What further C. Varicocele treatment tactics should be chosen? D. Dystopic kidney E. Necrotic papillitis A. B. 10. A 59-year-old man complains of pain C. Cold to the abdomen in his left eye and left side of his head, D. Ultrasound investigation significant vision impairment of the left eye, E. Laparocentesis nausea, and vomiting. Visual acuity of the right eye is 1.0. Visual acuity of the left 14. A 48-year-old woman has arrived to eye is 0.03, attempts at correction bring no the surgical unit with wounds in her thigh. improvement. Right eye intraocular pressure On examination the wound surface has - 21 mm Hg, left eye intraocular pressure - dirty-gray coating with unpleasant sweet 65 mm Hg. Congestive injection is observed smell. Wound content resembles raspberry on the sclera of the left eye. The cornea is jelly. Skin tissues around the wound are thick and swollen. The anterior chamber is glossy and turgid. Palpation reveals moderate shallow, moist, and clear. The pupil is dilated crepitation in the tissues. What microflora is and unresponsive to the light, the fundus of the most likely to cause such inflammation? the eye is not visible. What is the most likely diagnosis? A. Anaerobic clostridial B. Anaerobic non-clostridial A. Acute attack of glaucoma of the left eye C. Streptococci B. Acute iridocyclitis of the left eye D. Staphylococci C. Stage II intraocular tumor of the left eye E. Blue pus bacillus D. Endophthalmitis of the left eye E. Panophthalmitis of the left eye 15. After a pain attack in the right subcostal area, a 58-year-old woman with overnutrition 11. On the 15th day after a small trauma developed icteric skin and sclera, light- of the right foot, the patient developed colored feces, and dark urine. Her abdomen indisposition, fatigability, irritability, is distended and painful on palpation in the headache, elevated body temperature, and right subcostal area. Palpation detects liver sensation of constriction, tension, and enlargement by 2-3 cm. Blood test: total twitching in the muscles of the right shin. bilirubin - 90 mcmol/L, conjugated bilirubin What disease can be suspected? - 60 mcmol/L. What method of examination will be the most informative for diagnosis A. Tetanus clarification? B. Anaerobic gas gangrene C. Erysipelas D. Acute thrombophlebitis E. Thrombophlebitis of the popliteal artery 12. A patient has the second and third degree Хiрургiчний профiль 15

A. Retrograde cholangiopancreatography g/L, thrombocytopenia. Anamnesis states B. Intravenous cholegraphy that a similar condition was observed 1 year C. Infusion cholegraphy ago. Make the diagnosis: D. Percutaneous transhepatic cholegraphy E. US of the hepatopancreatobiliary zone A. Thrombocytopenic purpura B. Hemophilia 16. An 11-year-old boy for a month has been C. Ulcerative bleeding presenting with increasing pain in the right D. Rectal tumor femur. In the painful area there is a non- E. Nonspecific ulcerative colitis mobile painful tumor with unclear margins. The child complains of general indisposition, 20. A 30-year-old man came to the family weakness, increased body temperature up to physician. 2 months ago he underwent a 39oC. X-ray shows widened medullary cavity, surgery for open fracture of the humerus. small foci of cancellous bone destruction, and On examination the patient’s condition is onion-like lamellar exfoliation of the cortical satisfactory; in the area of the postoperative layer. What is the most likely pathology wound there is a fistula that discharges resulting in such clinical presentation? a small amount of pus; the area itself is red; fluctuation is detected. X-ray shows A. Ewing sarcoma destruction of the humerus with sequestra. B. Osteogenic sarcoma What complication did the patient develop C. Fibrosarcoma during the postoperative period? D. Chondrosarcoma E. Juxtacortical sarcoma A. Posttraumatic osteomyelitis B. Hematogenous osteomyelitis 17. A 43-year-old man complains of a C. Wound suppuration protrusion in the right inguinal region, D. Posttraumatic phlegmon that enlarges due to strain. He has been E. Suture sinus presenting with this condition for 6 months. Within this period the protrusion has grown. 21. 3 hours after a trauma, a young Objectively in the right inguinal region an man developed bradycardia of 46/min., elastic protrusion 8x5 cm is visible. On anisocoria D>S, hemi-hyperreflexia S>D, palpation it disappears, leaving an empty hemihypesthesia on the left, and a convulsive space 4x4 cm between the pedicles of disorder. The character of this process needs the Poupart ligament. ”Cough push” sign to be clarified. What method of examination is positive over this opening. Make the will be the most accurate for this purpose? diagnosis: A. Brain CT A. Right-sided reducible inguinal hernia B. Skull X-ray B. Right-sided reducible femoral hernia C. Electroencephalography C. Cyst of the right spermatic cord D. Echoencephalography D. Right-sided inguinal lymphadenitis E. Lumbar puncture E. Right-sided reducible arcuate line hernia 22. The body of a 24-year-old woman with 18. A 78-year-old man with a prostate suspected poisoning has been found on adenoma underwent a herniotomy for a the street. Forensic medical examination direct inguinal hernia. After the surgery he was requested by an investigator during presents with absent urination. Enlarged examination of the site and the body. urinary bladder is detectable above the According to the Criminal Procedure Code patient’s pubis. What measures should be currently in force in Ukraine, forensic taken in this case? medical examination is required when it is necessary to determine the: A. Bladder catheterization B. Apply cold to the urinary bladder area A. Cause of death C. Prescribe processing of the postoperative B. Manner of death wound with UHF field C. Time of death D. Prescribe proserin (neostigmine) D. Mode of death intramuscularly E. Mechanism of death E. Prescribe antispasmodics subcutaneously 23. A 37-year-old patient complains of pain 19. A 38-year-old patient has been brought in the spinal column, reduced mobility. The by an ambulance to the surgical department condition persists for 7 years. ”Sway back” with complaints of general weakness, is observed, there is no movement in all indisposition, black stool. On examination spinal regions. X-ray shows ”bamboo spine” the patient is pale, there are dotted vertebral column. What is the most likely hemorrhages on the skin of his torso and diagnosis? extremities. On digital investigation there are black feces on the glove. Blood test: Hb- 108 Хiрургiчний профiль 16

A. Ankylosing spondyloarthritis A. Acute appendicitis B. Osteochondrosis B. Right-sided renal colic C. Spondylitis deformans C. Cecal tumor D. Tuberculous spondylitis D. Intestinal obstruction E. Spondylolisthesis E. Acute cholecystitis 24. A surgery unit received a person with 28. A 45-year-old man diagnosed with acute an incised stab wound on the upper third pulmonary abscess suddenly developed of the right thigh. Examination detects an sharp pain in his chest on the right and incised stab wound 3.0x0.5x2.0 cm in size dyspnea up to 30/min. Examination detects on the inner surface of the upper third of facial cyanosis and shallow rapid respirations. the right thigh. Bright-red blood flows from Auscultation reveals acutely weakened deep within the wound in a pulsing stream. respiration throughout the whole right lung; Characterize this type of bleeding: percussion reveals a vesiculotympanitic (bandbox) resonance at the lung apex A. Arterial and dullness in the lower lobe. What B. Venous complication developed in this patient? C. Parenchimatous D. Capillary A. Pyopneumothorax E. Mixed B. Pleuropneumonia C. Pneumothorax 25. A 47-year-old man developed the D. Acute mediastinitis signs of decompensated laryngeal stenosis E. Esophageal perforation against the background of acute flegmonous laryngitis. He presents with inspiratory 29. A 5-year-old child was brought to the dyspnea at rest, forced position, cyanotic skin ENT department by an ambulance. The covered in cold sweat, tachycardia, deficient child presents with cough and difficult pulse, and low blood pressure. What urgent respiration. From the patient’s history it treatment tactics should be chosen? is known that the child was playing with a toy construction set, when suddenly A. Tracheostomy started coughing and developed labored B. Oral administration of hyposensitization breathing. Examination detects periodical substances and broncholytics cough, labored expiration, and respiratory C. Intravenous administration of dehydrating lag in the left side of the child’s thorax. agents Auscultation: diminished respiration on the D. Administration of glucocorticoid hormones left. Percussion: tympanitis. X-ray shows a E. Oxygen therapy displacement of the mediastinal organs to the right. Make the diagnosis: 26. Heart X-ray of a 31-year-old man has revealed the following: with tightly filled A. A foreign body in the left bronchus, opacified esophagus there is a marginal filling valvular bronchostenosis defect in its middle third on the posterior B. A foreign body in the right bronchus, wall; the defect is 1.8x1.3 cm in size with valvular bronchostenosis clear oval border. Mucosal folds are retained C. A foreign body in the trachea and envelop the defect; wall peristalsis D. A foreign body in the left bronchus, and elasticity are not affected. There are complete bronchostenosis no complaints regarding the condition of E. A foreign body in the right bronchus, the patient’s alimentary canal. Make the partial bronchostenosis provisional diagnosis: 30. A 30-year-old man was brought to the A. Esophageal tumor neurosurgical department with complaints B. Achalasia cardiae of constant headaches, nausea, vomiting, C. Esophageal burns fever, and weakness of the right-side limbs. D. Diverticulum Anamnesis states that one month ago E. Barrett esophagus the patient had a surgery for left-sided suppurative otitis and mastoiditis. He has 27. A 25-year-old man was hospitalized with been undergoing treatment in an ENT complaints of pain in his lower abdomen and department. Approximately 2 weeks ago right lumbar area that appeared one hour the temperature increased, and the patient ago. Patient’s general state is moderately oC developed headaches. Objectively: heart rate severe. Body temperature - 38.2 , heart rate - 98/min., BP- 140/90 mm Hg, temperature - - 102/min. The tongue is dry. The abdomen 38.3oC. Neurologically manifested stiff neck: is painful on deep palpation in the right bilateral Kernig’s symptom, unsteadiness iliac area and in the Petit triangle. Aure- during the Romberg’s maneuver. Computer Rozanov and Gabay signs are positive. Make tomography of the brain revealed a three- the provisional diagnosis: dimensional growth with a capsule in the left hemisphere. Make the diagnosis: Хiрургiчний профiль 17

sputum in the amount of 100 mL per A. Cerebral abscess day, hemoptysis, dyspnea, increased body B. Echinococcus temperature up to 37.8oC, general weakness, C. Hemorrhage weight loss. The patient’s condition lasts for D. Hydrocephalus 4 years. Exacerbations occur 2-3 times a E. Arnold-Chiari malformation year. The patient presents with malnutrition, pale skin, cyanosis of the lips, drumstick 31. The burns unit received a patient, who (clubbed) fingers. Tympanic percussion 6 hours ago during a fire received flame sound in the lungs, weakened respiration, burns. On the patient’s body there is gray- numerous various moist crackles in the lower brown area of necrosis that covers 3/4 of pulmonary segments on the left can be the body perimeter. Occasionally there are observed. In blood: erythrocytes - 3.2·1012/L, small blisters with hemorrhagic contents and 9 patches of shredded epidermis. What local leukocytes - 8.4·10 /L, ESR- 56 mm/hour. On therapy is necessary in this case? X-ray: lung fields are emphysematous, the left pulmonary root is deformed and dilated. A. Decompression necrectomy What is the most likely diagnosis? B. Chemical necrolysis C. Blister puncture A. Multiple bronchiectasis of the left lung D. Necrectomy with xenotransplantation B. Chronic left-sided pneumonia E. Necrectomy with dermal autograft C. Chronic abscess of the left lung D. Left-sided pulmonary cystic dysplasia 32. A woman in her early- to mid-thirties E. Suppuration of the cyst in the left lung has lost her consciousness 3-5 minutes ago. On examination: the skin is pale, no pulse 36. A 57-year-old woman during a regular over the carotid arteries, no spontaneous ultrasound examination presented with a respiration, pupils are dilated; the patient space-occupying heterogeneous lesion in the is nonresponsive, presents with atony. The right kidney. What is the most informative patient’s condition can be determined as: method of renal tumor diagnostics? A. Clinical death A. Spiral computed tomography B. Natural death B. Excretory urography C. Syncope C. Retrograde pyelography D. Brain death D. Radioisotope renography E. Comatose state E. Three glass urine test 33. A boy had a foreign body removed 37. A 40-year-old victim of a traffic accident from under his nail plate. 3 days later he sustained the following injuries: closed developed a sharp throbbing pain at the diaphyseal femur fracture, brain concussion, end of his distal phalanx, which intensifies multiple rib fractures, hemopneumothorax, when the phalanx is pressed, hyperemia of degloving shin injuries. What injuries require the nail fold, elevated body temperature up the most urgent attention? to 38.5oC, and nail plate discoloration. Make the diagnosis: A. Multiple rib fractures, hemopneumothorax B. Closed diaphyseal femur fracture A. Subungual panaritium C. Brain concussion B. Erysipelas D. Degloving shin injuries C. Paronychia E. All injuries are equivalent D. Erysipeloid E. Abscess 38. At the railroad crossing a passenger train collided with a bus. In this collision 26 34. A 32-year-old woman complains of body bus passenges died, another 18 passengers weight loss despite her increased appetite, received mechanical injuries of varying nervousness, and tremor of the extremities. severity. Where will be professional medical Objectively: the skin is moist; the thyroid aid provided for the victims of this accident? gland is diffusely enlarged, painless, soft, and Who will provide this aid? mobile. Blood test: increased level of T3, T4, and thyroid-stimulating hormone (THS). A. In medico-prophylactic institutions; What is the most likely diagnosis? general and surgeons B. At the site of the accident; first-response A. Diffuse toxic goiter emergency teams B. Thyroid carcinoma C. At the site of the accident; specialized C. Autoimmune (Hashimoto’s) thyroiditis second-response emergency teams D. Thyroid adenoma D. In medico-prophylactic institutions; E. Diffuse nontoxic goiter specialized second-response emergency teams E. In medical institutions; all listed types of 35. A 19-year-old young man complains healthcare workers of cough with expectoration of purulent Хiрургiчний профiль 18

39. A 45-year-old man underwent a cardiac ambulance into the emergency hospital. He surgery one week ago. His general state has complains of sudden pain in the lumbar area, been deteriorating since then: dyspnea at frequent painful urination, and vomiting. rest, retrosternal pain that irradiates to the Examination detects pain in the lumbar neck, marked weakness. Objectively his body area, costovertebral angle tenderness, pain temperature is hectic. His cardiac borders on palpation of kidneys and along the are expanded, apical beat is weakened. ureter on the right. Urine test: proteins, Auscultation detects pericardial friction rub. fresh erythrocytes, leukocytes. Make the What is the most likely diagnosis? provisional diagnosis: A. Acute pericarditis A. Urolithiasis, renal colic B. Acute cardiac aneurysm B. Acute pyelonephritis C. Myocardial infarction C. Acute glomerulonephritis D. Acute myogenic dilatation of the heart D. Acute renal failure E. Pulmonary embolism E. Polycystic kidney disease 40. A 45-year-old man was brought by an Педiатричний профiль 19

1. A newborn girl has Apgar score of 7-8 A. Doppler echocardiography points at the 1-5 minutes after birth. During B. Electrocardiography the labor there was a brief difficulty with C. Chest X-ray extraction of the shoulder girdle. After birth D. Rheography of the pulmonary artery the baby presents with disturbed function of E. Ultrasound of the liver the proximal segment and forced position of the right . The shoulder is rotated 5. Mother of a 5-year-old child noticed on the inwards, the elbow is extended, the forearm the head of her child a round ”bald” spot 3 is pronated, and the whole upper limb cm in diameter. All the hairs in the focus are resembles an arm of a doll. What is the most broken off at the length of 5-6 mm. The day likely clinical diagnosis in this case? before the child was petting a stray cat. Make the diagnosis: A. Erb-Duchenne palsy B. Thoracic spine trauma A. Microsporia C. Osteomyelitis of the right arm B. Superficial trichophytosis D. Intracranial hemorrhage C. Deep trichophytosis E. Soft tissue injury of the right arm D. Psoriasis E. Alopecia areata 2. Disease onset was acute. A child developed general weakness, pain in the 6. A 2-year-old child with persisting cough joints, and elevated temperature. Later these and subfebrile body temperature after a case signs became accompanied by itching skin of URTI developed dyspnea, cyanosis of rash manifested as erythematous spots 2- the nasolabial triangle, percussion dullness 5 mm in size. The rash gradually turned and weakened respiration in the lower lobe hemorrhagic. Large joints are painful and of the right lung, and a slight mediastinal swollen; pain attacks periodically occur in displacement to the left. What pulmonary the paraumbilical area; there are signs of pathology is likely to cause this clinical intestinal hemorrhage. What is the most presentation? likely diagnosis? A. Pleurisy A. Hemorrhagic vasculitis (Henoch-Schonlein B. Emphysema C. Pneumonia purpura) D. Atelectasis B. Scarlet fever E. Bronchitis C. Hemorrhagic meningoencephalitis D. Streptococcal impetigo 7. During examination a 4-month-old E. Rheumatism child with meningococcemia presents with acrocyanosis, cold extremities, tachypnea, 3. A 13-year-old girl for the last two weeks and thready pulse, blood pressure of 30/0 has been complaining of dyspnea and mm Hg, anuria, and sopor. What clinical shin and foot edemas that appear after a syndrome is it? physical exertion. In the morning the edemas significantly decrease. Clinical examination A. Toxic shock syndrome revealed enlarged liver and coarse systolic B. Neurotoxicosis murmur over the heart area. Blood test and C. Exicosis urinalysis are without changes. What is the D. Encephalic syndrome most likely cause of edemas in this child? E. Acute renal failure A. Heart failure 8. At night a 2-year-old child with B. Nephrotic syndrome upper respiratory tract infection suddenly C. Acute pyelonephritis developed dyspnea with labored inspiration. D. Angioneurotic edema Objectively the skin is pale, perioral cyanosis E. Hepatic cirrhosis and slight acrocyanosis are observed. Breathing is loud, respiration rate is 32/min. 4. A 7-year-old boy has severe pulmonary Jugular, supra- and infraclavicular fossae mucoviscidosis (cystic fibrosis). He retract during breathing. Respiration is complains of dyspnea and blood coarse on auscultation. Heart sounds are expectoration. Objectively he presents with clear and sonorous, heart rate is 120/min. lagging physical development, acrocyanosis, What condition was complicated by the hepatomegaly, drumstick fingers, and nail development of the upper respiratory tract plates resembling a ”clock face”. Provisional infection? diagnosis of chronic pulmonary heart disease is made. What examination would be the A. Stenosing laryngotracheitis most informative for diagnosis confirmation? B. Airway foreign body C. Obstructive bronchitis D. Bronchiolitis E. Bronchial asthma Педiатричний профiль 20

9. A 1-year-old child with a case of URTI headache. Clinical urinalysis: proteins - suddenly developed noisy respirations with 7.1g/L,leukocytes-1-2inthevision difficult inspiration, intercostal retractions, field, erythrocytes - 3-4 in the vision field. and barking cough on the 2nd night after During the course of treatment the edemas the disease onset. What is the most likely gradually dissipated, headache abated, diagnosis? diuresis normalized. Daily urine proteins - 3 g/L. Biochemical blood test: total protein - A. Stenosing laryngotracheobronchitis 43.2 g/L, urea - 5.2 mmol/L, cholesterol - 9.2 B. Acute pulmonary inflammation mmol/L. What glomerulonephritis syndrome C. Bronchial asthma is the most likely to be present in the patient? D. Acute bronchitis E. Acute bronchiolitis A. Nephrotic B. Nephritic 10. A 10-year-old boy with symptoms of C. Isolated urinary arthritis and myocarditis was brought to a D. Hematuric hospital. Based on clinical examination the E. Mixed provisional diagnosis of juvenile rheumatoid arthritis was made. What symptom is the 14. A 3-month-old child with signs of rickets most contributive for the diagnostics of this presents with positive Chvostek, Trousseau, disease? and Maslov signs. One day ago the parents witnessed a cyanotic attack in their child - A. Reduced mobility of the joints in the the child broke into a cold sweat, the eyes morning bulged, and respiratory arrest occurred. One B. Regional hyperemia of the joints minute later the child drew in a loud breath C. Affection of the large joints and the child’s condition normalized again. D. Enlarged heart What is the cause of the described signs of E. Increased heart rate the disease? 11. A 7-year-old girl has been twice treated A. Decrease of blood calcium levels with antibacterial agents for urinary tract B. Increase of blood calcium levels infection. US shows no severe renal defects. C. Decrease of blood phosphorus levels The child presents with recurrence of D. Increase of blood phosphorus levels leukocyturia and bacteriuria, elevated body E. Metabolic acidosis temperature up to 38.5oC, and pain in her left lumbar area. What examination should be 15. A newborn with gestational age of conducted first to clarify the cause of urinary 31 weeks presents with hypotonia and infection recurrence? depressed consciousness. Hematocrit is 35%, general cerebrospinal fluid analysis A. Micturating cystourethrography shows increased content of erythrocytes B. Excretory urography and protein, and low glucose. These data C. Retrograde pyelography correspond with the clinical presentation of: D. Immunogram E. Radioisotope renography A. Intracranial hemorrhage B. Meningitis 12. A child is 1 year old. After solid food C. Sepsis was introduced into the diet, within the D. Anemia last several months the child developed E. Intrauterine infection loss of appetite, diarrhea with large amount of feces, and occasional vomiting. Body 16. A newborn has Apgar score of 9. When temperature remains normal. Body weight should this infant be put to the breast? is 7 kg. The child is very pale, has leg edemas and extremely distended abdomen. Feces A. In the delivery room analysis detects high levels of fatty acids and B. After 12 hours soaps. Diagnosis of celiac disease was made C. After 2 hours and gluten-free diet was prescribed. What D. On the 2nd day should be excluded from the diet in this case? E. On the 3rd day A. Cereals - wheat, oats 17. A 3-week-old infant developed large, B. Milk and dairy products flaccid vesicles with purulent contents on C. Fruits the skin of chest and abdomen. The vesicles D. Animal protein rupture quickly. Make the provisional E. Easily digestible carbohydrates diagnosis: 13. A 7-year-old boy has been an inpatient for 1.5 months. He had been brought to the hospital with complaints of edemas all over his body, low urine output, and Педiатричний профiль 21

A. Pemphigus neonatorum A. Tetrad of Fallot B. Vesiculopustulosis B. Coarctation of the aorta C. Toxic erythema C. Transposition of the great vessels D. Pemphigus syphiliticus D. Ventricular septal defect E. Pseudofurunculosis E. Acquired valvular disease 18. 10 hours after birth a child developed 22. A 15-year-old girl complains of dizziness jaundice, hypotonia, hyporeflexia, and and sensation of lack of air that she develops moderate hepatosplenomegaly. Feces and in emotionally straining situations. Relief urine are of normal color. Umbilical cord occurs after she takes corvalol. Objectively: blood bilirubin is 51 mcmol/L due to hyperhidrosis and marble-like pattern of unconjugated bilirubin levels. In venous the skin of her palms and feet. Clinical blood: erythrocytes - 3.5·1012/L, Нb- 140 g/L, and instrumental examination revealed no reticulocytes - 1.5%, bilirubin - 111 mcmol/L, organic changes in the central nervous, conjugated - 11 mcmol/L, ALT- 40 U/L, AST- cardiovascular, and respiratory systems. 30 U/L. Mother’s blood group is А(II) Rh(- What provisional diagnosis can be made? ), child’s blood group is А(II) Rh(+). What A. Somatoform autonomic dysfunction laboratory test can confirm the diagnosis? B. Obstructive bronchitis A. Coombs test C. Bronchial asthma B. Viral hepatitis markers analysis D. Stenosing laryngotracheitis C. Measurement of erythrocyte osmotic E. Acute epiglottitis resistance 23. A 1.5-month-old child on breasfeeding D. Erythrocytometry presents from birth with daily vomiting, E. Measurement of glucose 6-phosphate irregular liquid foamy feces, and meteorism, dehydrogenase levels in erythrocytes which are resistant to antibacterial and 19. A 6-month-old child on breastfeeding probiotic therapy; no increase of body mass is hospitalized in the inpatient department. is observed. The child’s condition improved, After the child recovers, the doctor when breastmilk was substituted with ”NAN recommends the mother to start introducing low lactose” formula. What pathology is it? solid food to the child’s diet. What products A. Lactase deficiency should be introduced to the child’s diet first? B. Intestinal lambliasis (Giardiasis) A. Vegetable puree C. Infectious enteritis B. Fermented dairy products D. Drug-induced enteritis C. Grated apple E. Functional dyspepsia D. Semolina porridge E. Buckwheat porridge 24. A 13-year-old girl for a month has been complaining of fatigability, dull pain in her 20. The 5-year-old child has been ill for 2 right subcostal area, abdominal distension, weeks. Cough attacks developed first and and constipations. Abdominal palpation were then followed by reprises. During reveals positive Kehr, Murphy, and Ortner coughing the child’s face turns red and signs, while Desjardins and Mayo-Robson cervical veins bulge. The cough attacks points are painless. Total bilirubin is 14.7 induce vomiting. X-ray shows intensified mcmol/L, predominantly indirect, ALT- 20 bronchial pattern. Blood test: leukocytes - U/L, AST- 40 U/L, amylase - 6.3 mmol/L. 16 · 109/L , lymphocytes - 72%, erythrocyte Echocholecystography shows practically no sedimentation rate - 4 mm/hour. What is the contraction of the gallbladder. Make the most likely diagnosis? provisional diagnosis: A. Pertussis A. Hypokinetic biliary dyskinesia B. Obstructive bronchitis B. Hyperkinetic biliary dyskinesia C. Pneumonia C. Chronic pancreatitis D. Adenovirus infection D. Acute pancreatitis E. Foreign body E. Chronic hepatitis 21. A 3-year-old child presents with 25. A 22-day-old infant developed dyspnea that abates in the sitting position, subcutaneous red nodes from 1.0 to 1.5 cm in size on the scalp; later the nodes suppurated. occasional loss of consciousness and seizures, o delayed physical development, cyanosis, Temperature increased up to 37.7 C, drumstick fingers. Echocardioscopy detects intoxication symptoms appeared, regional aortic dextraposition, ventricular septal lymph nodes enlarged. Complete blood defect, pulmonary artery stenosis, and right count: anemia, leukocytosis, neutrocytosis, ventricular hypertrophy. What is the most increased ESR. What diagnosis can be made? likely diagnosis? Педiатричний профiль 22

A. Pseudofurunculosis Objectively her skin is pale and cold to touch, B. Pemphigus her pupils are dilated. Blood pressure - 90/50 C. Vesiculopustulosis mm Hg. Heart rate - 58/min. What pathology D. Scalp phlegmon occurred in this case? E. - A. Syncope 26. A 10-year-old boy was brought into B. Sympathicotonic collapse the hospital with complaints of expiratory C. Paralytic collapse dyspnea, respirations are 30/min. He explains D. Sunstroke his state by a change in the weather E. - conditions. For the last 4 years the boy has been registered for regular check-ups due 29. A 13-year-old girl has 30% of excessive to his diagnosis of third degree persistent body mass, she started to gain weight at bronchial asthma. To provide emergency aid the age of 3. She has a family history of forthischild,first he needs to be given: obesity. Her height and sexual development are normal for her age. The appetite is A. Salbutamol or short-acting β2-agonists excessive. She complains of periodical B. Dexamethasone headaches. Blood pressure - 120/80 mm Hg. C. Adrenaline Subcutaneous fat is evenly distributed, she D. Euphylline (Aminophylline) has no stretch marks. There is juvenile acne E. Claritin (Loratadine) on her face. What type of obesity is it? 27. A 3-year-old child has been brought to A. Alimentary constitutive obesity a hospital with complaints of pain in the B. Hypothalamic obesity legs, fever, and loss of appetite. Objectively: C. Adrenal obesity pale skin and mucosa, hemorrhagic rash. D. Hypothalamic syndrome of puberty Lymph nodes are enlarged, painless, dense E. Hypothyroid obesity and elastic, not matted together. Bones, joints, and abdomen are painful. The liver 30. An 8-year-old girl complains of frequent and spleen are enlarged. Hemogram: Hb- 88 painful urination in small amounts and g/L, color index - 1.3, platelets - 80 · 109/L, urinary incontinence. The signs have leukocytes - 25.8 · 109/L, lymphoblasts - 70%, been present for 2 days already. She ESR- 52 mm/hour. Make the provisional explains her disease by overexposure to diagnosis: cold. Costovertebral angle tenderness is absent. Complete blood count is without A. Acute leukemia pathologies. Urine test: leukocytes - 20-30 B. Thrombocytopenic purpura in the vision field, erythrocytes - 40-50 in the C. Acute rheumatic fever vision field, unchanged, bacteriuria. What is D. Infectious mononucleosis the most likely diagnosis? E. Hemorrhagic vasculitis (Henoch-Schonlein purpura) A. Cystitis B. 28. During an outdoors school event in C. Pyelonephritis hot weather, a 10-year-old girl lost her D. Glomerulonephritis consciousness. Body temperature - 36.7oC. E. Urolithiasis Акушерство i гiнекологiя 23

1. A pregnant woman is 28 years old. irritability, and headaches. Her skin is Anamnesis: accelerated labor complicated of normal color. Blood pressure - 150/90 by the II degree cervical rupture. The mm Hg, pulse - 90/min., rhythmic. The following two pregnancies resulted in abdomen is soft and painless. Bimanual spontaneous abortions at the terms of 12 and examination shows no uterine enlargement, 14 weeks. On mirror examination: the uterine the appendages cannot be detected. The is scarred from previous ruptures at vaginal fornices are free. What is the most 9 and 3 hours, the cervical canal is gaping. likely diagnosis? On vaginal examination: the cervix is 2 cm long, the external orifice is open 1 cm wide, A. Climacteric syndrome the internal orifice is half-open; the is B. enlarged to the 12th week of pregnancy, soft, C. Adrenogenital syndrome mobile, painless, the appendages are without D. Stein-Leventhal syndrome (polycystic changes. What diagnosis can be made? syndrome) E. Uterine myoma A. Isthmico-cervical insufficiency, habitual noncarrying of pregnancy 5. A 30-year-old multigravida has been B. Threatened spontaneous abortion in labour for 18 hours. 2 hours ago the C. Incipient abortion, habitual noncarrying of pushing stage began. Fetal heart rate is clear, pregnancy rhythmic, 136/min. Vaginal examination D. Cervical hysteromyoma, habitual reveals complete cervical dilatation, the fetal noncarrying of pregnancy head in the pelvic outlet plane. Sagittal E. Cervical pregnancy, 12 weeks suture is in line with obstetric conjugate, the occipital fontanel is near the pubis. The 2. On the day 4 after the cesarean section patient has been diagnosed with primary a woman developed fever with body uterine inertia. What is the further tactics temperature up to 39oC and abdominal of labor management? pain. Pulse - 104/min. She vomited twice. The patient is sluggish, her tongue is dry A. Outlet forceps and has gray coating. The abdomen is B. Labour stimulation distended. Signs of peritoneal irritation are C. Cesarean section positive in all segments. Peristalsis cannot D. Skin-head Ivanov’s forceps be auscultated. No passage of gas occurs. E. Vacuum extraction of the fetus Uterine fundus is located at the level of the navel. The uterus is painful on palpation. The 6. A woman is 40 weeks pregnant. discharge is moderate and contains blood The fetus is in the longitudinal lie and and pus. What is the most likely diagnosis? cephalic presentation. Pelvic size: 26-29-31- 20. Expected weight of the fetus is 4800 gram. A. Diffuse peritonitis The labor contractions has been lasting for B. Metroendometritis 12 hours, within the last 2 hours they were C. Progressive thrombophlebitis extremely painful, the parturient woman D. Pelvic peritonitis is anxious. The waters broke 4 hours ago. E. On external examination the contraction ring is located 2 finger widths above the 3. A 58-year-old woman came to the navel, Henkel-Vasten sign is positive. Fetal gynecological clinic. She complains of heart rate is 160/min., muffled. On internal bloody discharge from her genital tracts. examination the uterine cervix is fully open, Menopause is 8 years. Gynecological the head is engaged and pressed to the examination: the uterus is slightly enlarged, entrance into the lesser . What is the dense to touch, with limited mobility; the most likely diagnosis? cannot be detected; is free. Fractional curettage of A. Threatened uterine rupture the uterine cavity yields a significant amount B. Complete uterine rupture of medullary substance in the scrape. What is C. Hyperactive uterine contractions the most likely diagnosis? D. Abruption of the normally positioned placenta A. Uterine corpus cancer E. Anatomically contracted pelvis B. C. Chorioepithelioma 7. A 23-year-old woman came to the D. Uterine cervix cancer gynecological clinic. She complains of pain, E. Hormone-producing ovarian tumor itching, and burning in her vulva, general weakness, indisposition, elevated body 4. A 48-year-old woman complains of temperature up to 37.2oC, and headache. On disturbed : her periods last examination in the vulva there are multiple for 7-9 days and are excessively profuse vesicles up to 2-3 mm in diameter with throughout the last half-year. She notes clear contents against the background of occasional hot flashes in her head, insomnia, hyperemia and mucosal edema. Make the Акушерство i гiнекологiя 24 provisional diagnosis: discharge is bloody and profuse. What is the most likely diagnosis? A. Genital herpes infection B. Primary syphilis A. 12-week pregnancy, spontaneous abortion C. Papillomavirus infection in progress D. Vulvar cancer B. 12-week pregnancy, threatened E. Cytomegalovirus infection spontaneous abortion C. Disturbed menstrual cycle, 8. A woman with the pregnancy term of 8 hyperpolymenorrhea weeks complains of elevated temperature up Disturbed menstrual cycle, o D. to 37.6 C, skin rash that can be characterized E. Full-term pregnancy, term labor as macular exanthema, enlargement of posterior cervical and occipital lymph 12. A 30-year-old woman came to the nodes, small amount of bloody discharge gynecological department. She complains from the genital tracts. She was examined of sharp pain in her lower abdomen and by the infectious diseases specialist and temperature of 38.8oC. She has a history of diagnosed with rubella. What tactics should extramarital sexual activity and 2 artificial the obstetrician-gynecologist choose? abortions. On gynecological examination the uterus is unchanged. The appendages A. Abortion are bilaterally enlarged and painful. Profuse B. Prescription of antibacterial therapy purulent discharge is being produced from C. Prescription of antiviral therapy the vagina. What examination needs to be D. Treatment of incipient abortion conducted to clarify the diagnosis? E. Prescription of hemostatic therapy A. Bacteriological and bacterioscopic analysis 9. A 16-year-old girl has primary B. Hysteroscopy amenorrhea, no pubic hair growth, normally C. Curettage of the uterine cavity developed mammary glands; her genotype is D. Colposcopy 46 ХY; uterus and vagina are absent. What is E. Laparoscopy your diagnosis? 13. It is the 3rd day after the first normal A. Testicular feminization syndrome term labor; the infant is rooming-in with the B. Mayer-Rokitansky-Kuster-Hauser mother and is on breastfeeding. Objectively: syndrome the mother’s general condition is satisfactory. C. Cushing syndrome Temperature is 36.4oC, heart rate is 80/min., D. Sheehan syndrome BP is 120/80 mm Hg. Mammary glands E. Cushing disease are soft and painless; lactation is moderate, unrestricted milk flow. The uterus is dense, 10. A 46-year-old woman came to the the uterine fundus is located 3 finger widths maternity clinic with complaints of moderate below the navel. Lochia are sanguino-serous, blood discharge from the vagina, which moderate in volume. Assess the dynamics of developed after the delay of 1.5 uterine involution: months. On vaginal examination: the cervix is clean; the uterus is not enlarged, mobile, A. Physiological involution painless; appendages without changes. Make B. Subinvolution the diagnosis: C. Lochiometra D. Pathologic involution A. Dysfunctional uterine bleeding E. B. Adenomyosis C. Ectopic pregnancy 14. A 28-year-old woman complaining of D. Submucous uterine myoma irregular and came E. Cancer of the uterine body to the gynecological clinic. Menstruations occur since the age of 15, irregular, with 11. A 25-year-old woman was brought into delays up to 2 months. On examination she the gynecological department with profuse presents with marked hirsutism and excessive bloody discharge from her genital tracts. body weight. On vaginal examination the She is 12 weeks pregnant, the pregnancy uterus is reduced in size and painless. is planned. Within the last 3 days she was The on the both sides are dense experiencing pains in her lower abdomen and enlarged. Ultasound shows microcystic that eventually started resembling cramps, changes in the ovaries, the ovaries are she developed bleeding. Her skin is pale, 5х4 cm and 4.5х4 cm in size with dense pulse - 88/min., blood pressure - 100/60 mm o ovarian capsule. Basal body temperature Hg, body temperature - 36.8 C. Vaginal is monophasic. What is the most likely examination: the uterus size corresponds diagnosis? with 11 weeks of pregnancy, the cervical canal allows inserting 1 finger and contains fragments of the fertilized ovum, the Акушерство i гiнекологiя 25

A. Polycystic ovary syndrome complication occurred in the patient? B. Krukenberg tumor C. Endometrioid cysts A. Posthemorrhagic anemia D. Bilateral adnexitis B. Somatoform autonomic dysfunction of E. Ovarian carcinoma hypotonic type C. Migraine 15. An 18-year-old girl was brought into the D. Gastritis gynecology deparment with complaints of E. elevated body temperature up to 37.8oC, sharp pain in her lower abdomen, more 19. A 22-year-old postparturient woman on intense on the right, and difficult defecation. the 12th day after the normal childbirth Vaginal examination detected a painful informs of elevated body temperature up to dense elastic formation 5x6 cm in the area 39oC for the last 3 days and pain in her right of her right ovary. Pregnancy test is negative. mammary gland. The right mammary gland What is the most likely diagnosis? is enlarged, hot to touch, tense, hyperemic, and painful. Palpation reveals there a dense A. Torsion of ovarian tumor pedicle infiltration 8x8 cm with a fluctuation in its B. Ectopic pregnancy center. What is the most likely diagnosis? C. Appendicitis D. rupture A. Postpartum period, day 12. Right-sided E. Ovarian apoplexy infiltrative-purulent mastitis B. Postpartum period, day 12. Right-sided 16. A 26-year-old woman presents with serous mastitis amenorrhea. 10 months ago she gave birth C. Postpartum period, day 12. Right-sided for a second time. In her early postpartum gangrenous mastitis period she developed a massive hypotonic D. Postpartum period, day 12. Right-sided hemorrhage. No breasfeeding. Lately she phlegmonous mastitis has been presenting with loss of weight, E. Postpartum period, day 12. Right-sided loss of hair, and indisposition. Gynecological lactostasis examination revealed atrophy of the external genitals, the uterus is abnormally small, no 20. A 35-year-old pregnant woman with uterine appendages can be detected. What is degree 1 essential hypertension, developed the most likely diagnosis? edemas and headache at the 33 week of her pregnancy. Objectively her general condition A. Sheehan syndrome (postpartum pituitary is satisfactory, blood pressure - 160/100 mm gland necrosis) Hg, normal uterine tone. Fetal heart rate B. Physiological amenorrhea is 140/min., rhythmic. She was diagnosed C. Suspected progressing ectopic pregnancy with daily proteinuria - 4 g/L, daily diuresis D. Stein-Leventhal syndrome (polycystic - 1100 mL. Creatinine - 80 mcmol/L, urea ovary syndrome) - 7 mmol/L, platelets - 100 · 109/L. What E. Galactorrhea-amenorrhea syndrome complication of pregnancy occurred? 17. A 45-year-old woman came to the A. Moderate preeclampsia maternity clinic with complaints of periodical B. Severe preeclampsia pains in her mammary glands that start C. Mild preeclampsia 1 day before menstruation and stop after D. Hypertensive crisis the menstruation begins. Palpation of the E. Renal failure mammary glands detects diffuse nodes predominantly in the upper outer quadrants. 21. A 24-year-old pregnant woman on What is the most likely diagnosis? her 37th week of pregnancy has been brought to the maternity obstetric service A. Fibrocystic mastopathy with complaints of weak fetal movements. B. Breast cancer Fetal heartbeats are 95/min. On vaginal C. Mastitis examination the uterine cervix is tilted D. Hyperprolactinemia backwards, 2 cm long, external orifice allows E. Breast cyst inserting a fingertip. Biophysical profile of the fetus equals 4 points. What tactics of 18. A 14-year-old girl came to the general pregnancy management should be chosen? practitioner with complaints of weakness, loss of appetite, headache, rapid fatigability. Her last menstruation was profuse and lasted for 14 days after the previous delay of 2 months. Objectively: the skin is pale, heart rate is 90/min., BP is 110/70 mm Hg, Hb is 88 g/L. Rectal examination: the uterus and its appendages are without changes, no discharge from the genital tracts. What Акушерство i гiнекологiя 26

A. Urgent delivery via a cesarean section 24. A 27-year-old woman complains of B. Treatment of placental dysfunction and foul-smelling discharge from her genital repeated analysis of the fetal biophysical tracts, pain in her lower abdomen, and profile on the next day elevated temperature. The complaints arose C. Doppler measurement of blood velocity in 2 days ago. She has a history of surgical the umbilical artery abortion at the term of 8 weeks one week D. Urgent preparation of the uterine cervix ago. Mirror examination: the uterine cervix for delivery is clear, external orifice produces foul- E. Treatment of fetal distress; if ineffective, smelling discharge. Vaginal examination: the then elective cesarean section on the next day uterus lies in anteflexion, is mobile, painful, and slightly enlarged. The appendages are 22. During regular preventive gynecological without changes. Make the provisional examination a 30-year-old woman was diagnosis: detected to have dark blue punctulated ”perforations” on the vaginal portion of A. Postabortal the uterine cervix. The doctor suspects B. Enterocolitis of the vaginal portion of the C. Appendicitis uterine cervix. What investigation method D. Acute respiratory disease would be most informative for diagnosis E. Salpingoophoritis confirmation? 25. A 17-year-old girl has made an A. Colposcopy, target biopsy of the cervix appointment with the doctor. She plans to B. US of the lesser pelvis begin her sex life. No signs of gynecological C. Hysteroscopy pathology were detected. In the family D. Curettage of the uterine cavity history the patient’s grandmother had E. Hormone testing cervical cancer. The patient was consulted about the maintenance of her reproductive 23. A parturient woman is 30 years old, stage health. What recommendation will be the I of the labor is ongoing. The fetus is in most helpful for prevention of invasive the cephalic presentation. Auscultation of cervical cancer? the fetal heart sounds detects bradycardia. Evaluation of cardiotocogram yielded the A. Vaccination against human papillomavirus following data: decrease of basal heart rate (HPV) down to 90/min., variability - monotonous (2 B. Vitamins, calcium, omega-3 and less); late decelerations with amplitude C. Immunomodulators of 50/min. Make the diagnosis and choose D. Antiviral and antibacterial drugs the obstetrical tactics necessary in this case: E. Timely treatment of sexually transmitted diseases A. Fetal distress. Urgent cesarean section delivery B. Fetal distress. Vacuum extraction delivery C. Normal condition of the fetus. Vaginal birth D. Fetal distress. Stimulation of uterine contractions E. Fetal distress. Forceps delivery Гiгiєна та органiзацiя охорони здоров’я 27

1. A 38-year-old woman works in flax processing, she dries flax. She came to the A. Endocrine disorders hospital complaining of difficult breathing, B. Acceleration constricting sensation in her chest, and cough C. Certain components of her diet attacks. These signs appear on the first day of D. Sports training her working week and gradually diminish on E. Deficient hygienic education the following days. What respiratory disease is likely in this case? 6. Employees work in conditions of high dust concentration. Certain chemical (silicon A. Byssinosis dioxide content) and physical properties of B. Silicosis dust aerosols contribute to the development C. Allergic rhinopharyngitis of occupational dust-induced diseases. What D. Bronchial asthma is the main physical property of dust E. Asthmatic bronchitis aerosols? 2. A district doctor has diagnosed one of A. Dispersion his patients with dysentery. What accounting B. Magnetization document reflects this type of morbidity? C. Electric charge D. Solubility A. Urgent report E. Ionization B. Statistical report C. Report on a major non-epidemic disease 7. In the factory cafeteria there was D. Certificate of temporary disability an outbreak of food poisoning. Clinical E. Control card of a patient registered for presentation indicates staphylococcal regular check-ups etiology of this disease. 15 people are sick. To c o n firm the diagnosis of food poisoning, 3. A 39-year-old man, a battery attendant, samples need to be sent to the laboratory. suddenly developed weakness, loss of What samples should be obtained for appetite, nonlocalized colicky abdominal analysis? pains, and nausea. Objectively his skin is gray; there is a pink-gray stripe on his gums; A. Vomit masses the stomach is soft and sharply painful. Blood B. Blood for hemoculture test detected erythrocytes with basophilic C. Blood (complete blood count) stippling and anemia. The patient has a D. Urine history of peptic ulcer disease of the stomach. E. Saliva Constipation tends to occur every 3-4 days. What is the most likely provisional 8. In April during the medical examination diagnosis? of various population groups, 27% of individuals presented with low working A. Saturnism (lead poisosning) ability and rapid fatigability. The following B. Acute appendicitis symptoms were observed in the affected C. Perforation of gastric ulcer individuals: swollen friable gingiva that D. Acute cholecystitis bleeds when pressed, hyperkeratosis E. Chronic alcoholism follicularis not accompanied by skin dryness. These symptoms most likely result from the 4. A 9-month-old infant presents with following pathology: delayed tooth eruption and fontanel closure, weakness, and excessive sweating. What type A. C-hypovitaminosis of hypovitaminosis is the most likely in this B. Parodontosis child? C. A-hypovitaminosis D. B1-hypovitaminosis A. Hypovitaminosis D E. Polyhypovitaminosis B. Hypovitaminosis C C. Hypovitaminosis B1 9. Increased general morbidity of the D. Hypovitaminosis B6 local population is observed in the area E. Hypovitaminosis A near a factory, where atmosphere is being intensively polluted with sulfurous gas. What 5. A 10-year-old girl exhibits high level of effect does polluted air have on human body physical development ( +3σ), her body in this case? length increased by 10 cm within a year (which is double the norm for her age group), A. Chronic nonspecific the number of permanent teeth corresponds B. Acute specific with the age norm (20), the development C. Acute nonspecific of her secondary sex characteristics is D. Chronic specific three years ahead of her age (Ма, Р, Ах, E. Selective Menarche). Development rate ahead of her 10. During medical examination of high and biological age can occur due to: Гiгiєна та органiзацiя охорони здоров’я 28 middle school students, the doctors vere sample there is 1 dead trichinella detected assessing correlation between biological and in 24 sections. This meat should be: calendar age of the school students based on the following criteria: height growth rate A. Sent for technical disposal per year, ossification of the carpal bones, the B. Allowed for sale with no restrictions number of permanent teeth. What additional C. Processed and sold through public catering development criterion should be assessed at network this age? D. Processed for boiled sausage production E. Frozen until the temperature of -10oC is A. Development of secondary sex reached in the deep layers, with subsequent characteristics exposure to cold for 15 days B. Body mass C. Chest circumference 16. To assess the effectiveness of medical D. Vital capacity of lungs technologies and determine the power and E. Hand strength direction of their effect on the indicators, the research was conducted to 11. During analysis of morbidity in the study the immunization rate of children and city, it was determined that age structure measles incidence rate by district. What of population is different in each district. method of statistical analysis should be What statistical method allows to exclude applied in this case? this factor, so that it would not skew the morbidity data? A. Calculation of correlation coefficient B. Calculation of morbidity index among the A. Standardization nonvaccinated B. Wilcoxon signed-rank test C. Calculation of matching factor C. Correlation-regression analysis D. Calculation of standardized ratio D. Dynamic time series analysis E. Calculation of statistical significance of the E. Analysis of average values difference between two estimates 12. Clinical statistical investigation was 17. Having studied the relationship between performed to determine effectiveness of the distance from villages to the local a new pharmacological preparation for outpatient clinics and frequency of visits to patients with ischemic heart disease. What the clinics among the rural population of parametric test (coefficient) can be used to this area, it was determined that the rank estimate the reliability of the results? correlation coefficient in this case equals -0.9. How can this relationship be characterized? A. Student’s t-distribution B. Sign test A. Strong inverse relationship C. Matching factor B. Strong direct relationship D. Wilcoxon signed-rank test C. Moderate inverse relationship E. Kolmogorov-Smirnov test D. Moderate direct relationship E. - 13. In a care area there is an increasing cervical cancer morbidity 18. In the inpatient gynecological unit within observed. The decision is made to conduct a year 6500 women underwent treatment. a medical examination of the women living They spent there a total of 102000 bed-days. in this locality. What type of medical What indicator of the gynecological unit examination is it? work can be calculated based on these data? A. Target A. Average length of inpatient stay B. Preliminary B. Average bed occupancy rate per year C. Regular C. Number of beds by hospital department D. Complex D. Bed turnover rate E. Screening E. Planned bed occupancy rate per year 14. In the process of hiring, a prospective 19. A middle school teacher with 4-year- employee has undergone preventive medical long record of work was issued a medical examination and was declared fittoworkin certificate for pregnancy and childbirth leave. this manufacturing environment. What type What amount of pay will she receive for the of preventive medical examination was it? duration of her leave in this case? A. Preliminary A. 100% of average salary B. Scheduled B. 50% of average salary C. Periodical C. 70% of average salary D. Specific D. 60% of average salary E. Comprehensive E. 80% of average salary 15. On laboratory investigation of a pork 20. In the air of the feed kitchen at the Гiгiєна та органiзацiя охорони здоров’я 29 poultry factory, at the area where formula feed is being mixed, the dust concentration A. Primary healthcare reaches 200 mg/m3.Airmicroflora is B. Emergency aid represented predominantly by Asperqillus C. Secondary healthcare and Mucor fungi. What effect determines D. Tertiary healthcare pathogenic properties of the dust? E. A. Allergenic 23. Human body receives from the B. Teratogenic atmosphere a number of chemicals. What C. Mutagenic type of action results in the combined effect D. Fibrogenic that is less than the sum of isolated effects of E. To x i c these chemicals on the body? 21. During regular medical examination a A. Antagonism lyceum student presents with signs of cheilitis B. Potentiation that manifests as epithelial maceration in C. Isolated action the area of seal. The lips are bright- D. Synergistic action red, with single vertical cracks covered with E. Complex action brown-red scabs. These clinical signs are 24. Clinical trials have proved the most likely caused by insufficient content of ”Lipoflavon”drugtobeeffectivefor the following in the diet: treatment of unstable angina pectoris in the A. Riboflavin control group and experimental group of B. Ascorbic acid patients. Neither patients nor researchers C. Retinol knew who belonged to which group. Name D. Thiamine this type of study: E. Calciferol A. Double blind study 22. A 30-year-old woman made an B. Simple blind study appointment with the family doctor for C. Triple-blind study scheduled vaccination of her 2-year-old D. Total-blind study child. What type of healthcare provides such E. Multicenter study medical services?